Given that \(a,b,c> 0\) and \(a+b+c=ab+bc+ca\).Prove that: \[\sum \frac{a^2}{a^2-a+1}\leq 3\]
Let x;y;z>0 . Prove that xyz+2(x2+y2+z2)+8≥5(x+y+z)
Revised version,find all the constant k such that xyz+k(x2+y2+z2)+8≥(k+3)(x+y+z) for all x;y;z>0
Let x;y;z>0 Prove that (x+y+z)2(x2+y2)(y2+z2)(z2+x2)≥8(x2y2+y2z2+z2x2)2
Let 0≤a;b;c≤1 . Put x=1−a+ab;y=1−b+bc;z=1−c+ca
a) Prove that x+y+z≥2
b)Prove that x2+y2+z2≥23
c)Prove or disprove x3+y3+z3≥45
Let be given positive integer n. Find the least real number k such that (xy)k(xn+yn)≤2 for all positive real numbers x;y satisfying the condition x+y=2
These problems has been taken from Vietnam TST practice for Hong Kong IMO 2016.
#Algebra
Easy Math Editor
This discussion board is a place to discuss our Daily Challenges and the math and science related to those challenges. Explanations are more than just a solution — they should explain the steps and thinking strategies that you used to obtain the solution. Comments should further the discussion of math and science.
When posting on Brilliant:
*italics*
or_italics_
**bold**
or__bold__
paragraph 1
paragraph 2
[example link](https://brilliant.org)
> This is a quote
\(
...\)
or\[
...\]
to ensure proper formatting.2 \times 3
2^{34}
a_{i-1}
\frac{2}{3}
\sqrt{2}
\sum_{i=1}^3
\sin \theta
\boxed{123}
Comments
There are no comments in this discussion.